Solving a differential equation with the Dirac-Delta function without Laplace transformations












6












$begingroup$


So I'm trying to solve the following differential equation:
$y''+3y'+2y=delta(t-1)$, $y(0)=0$, $y'(0)=0$. (where $delta$ is the Dirac's delta function)
Everything I've read in my textbook/online has solved these types of equations by taking the Laplace transformation, but our class hasn't covered Laplace transformations yet...anyone have any idea what I should do?










share|cite|improve this question











$endgroup$








  • 1




    $begingroup$
    What did you go over in class - have you even seen these sorts of differential equations before? Were you assigned some reading material preceding the homework? Are these textbook questions that lie at the end of a chapter? It's perplexing that you'd be expected to solve an equation but not expected to learn about how to do it first...
    $endgroup$
    – anon
    Oct 7 '11 at 3:36










  • $begingroup$
    Well, in class we did this example: y''-y=delta(t-3). The homogeneous solutions were y=e^-t and y=e^t. He then deduced that the particular solution would be: [(e^t)/2]*integral from 0 to t ( (e^-s)*delta(s-3)ds ) - [e^(-t)/2]*integral from 0 to t( (e^s)*delta(s-3))ds From there, he concluded that y(t) would be e^(t-3)/2 - e^(3-t)/2 for all t>3. I guess what I'm confused on is how much of the partucluar solution came from the homogenous, and how much is just always going to be part of the particular solution
    $endgroup$
    – ben
    Oct 7 '11 at 3:53








  • 1




    $begingroup$
    And does the way he 'then deduced' the particular solution have anything to do with formulas you already have? How did he justify it to the class if not with either a derivation or a formula you're expected to know?
    $endgroup$
    – anon
    Oct 7 '11 at 4:02










  • $begingroup$
    Dear ben, you can use the formula given in this answer.
    $endgroup$
    – Pierre-Yves Gaillard
    Oct 7 '11 at 15:36


















6












$begingroup$


So I'm trying to solve the following differential equation:
$y''+3y'+2y=delta(t-1)$, $y(0)=0$, $y'(0)=0$. (where $delta$ is the Dirac's delta function)
Everything I've read in my textbook/online has solved these types of equations by taking the Laplace transformation, but our class hasn't covered Laplace transformations yet...anyone have any idea what I should do?










share|cite|improve this question











$endgroup$








  • 1




    $begingroup$
    What did you go over in class - have you even seen these sorts of differential equations before? Were you assigned some reading material preceding the homework? Are these textbook questions that lie at the end of a chapter? It's perplexing that you'd be expected to solve an equation but not expected to learn about how to do it first...
    $endgroup$
    – anon
    Oct 7 '11 at 3:36










  • $begingroup$
    Well, in class we did this example: y''-y=delta(t-3). The homogeneous solutions were y=e^-t and y=e^t. He then deduced that the particular solution would be: [(e^t)/2]*integral from 0 to t ( (e^-s)*delta(s-3)ds ) - [e^(-t)/2]*integral from 0 to t( (e^s)*delta(s-3))ds From there, he concluded that y(t) would be e^(t-3)/2 - e^(3-t)/2 for all t>3. I guess what I'm confused on is how much of the partucluar solution came from the homogenous, and how much is just always going to be part of the particular solution
    $endgroup$
    – ben
    Oct 7 '11 at 3:53








  • 1




    $begingroup$
    And does the way he 'then deduced' the particular solution have anything to do with formulas you already have? How did he justify it to the class if not with either a derivation or a formula you're expected to know?
    $endgroup$
    – anon
    Oct 7 '11 at 4:02










  • $begingroup$
    Dear ben, you can use the formula given in this answer.
    $endgroup$
    – Pierre-Yves Gaillard
    Oct 7 '11 at 15:36
















6












6








6


1



$begingroup$


So I'm trying to solve the following differential equation:
$y''+3y'+2y=delta(t-1)$, $y(0)=0$, $y'(0)=0$. (where $delta$ is the Dirac's delta function)
Everything I've read in my textbook/online has solved these types of equations by taking the Laplace transformation, but our class hasn't covered Laplace transformations yet...anyone have any idea what I should do?










share|cite|improve this question











$endgroup$




So I'm trying to solve the following differential equation:
$y''+3y'+2y=delta(t-1)$, $y(0)=0$, $y'(0)=0$. (where $delta$ is the Dirac's delta function)
Everything I've read in my textbook/online has solved these types of equations by taking the Laplace transformation, but our class hasn't covered Laplace transformations yet...anyone have any idea what I should do?







ordinary-differential-equations laplace-transform






share|cite|improve this question















share|cite|improve this question













share|cite|improve this question




share|cite|improve this question








edited Oct 7 '11 at 9:03









J. M. is not a mathematician

61.5k5152290




61.5k5152290










asked Oct 7 '11 at 3:28









benben

3112




3112








  • 1




    $begingroup$
    What did you go over in class - have you even seen these sorts of differential equations before? Were you assigned some reading material preceding the homework? Are these textbook questions that lie at the end of a chapter? It's perplexing that you'd be expected to solve an equation but not expected to learn about how to do it first...
    $endgroup$
    – anon
    Oct 7 '11 at 3:36










  • $begingroup$
    Well, in class we did this example: y''-y=delta(t-3). The homogeneous solutions were y=e^-t and y=e^t. He then deduced that the particular solution would be: [(e^t)/2]*integral from 0 to t ( (e^-s)*delta(s-3)ds ) - [e^(-t)/2]*integral from 0 to t( (e^s)*delta(s-3))ds From there, he concluded that y(t) would be e^(t-3)/2 - e^(3-t)/2 for all t>3. I guess what I'm confused on is how much of the partucluar solution came from the homogenous, and how much is just always going to be part of the particular solution
    $endgroup$
    – ben
    Oct 7 '11 at 3:53








  • 1




    $begingroup$
    And does the way he 'then deduced' the particular solution have anything to do with formulas you already have? How did he justify it to the class if not with either a derivation or a formula you're expected to know?
    $endgroup$
    – anon
    Oct 7 '11 at 4:02










  • $begingroup$
    Dear ben, you can use the formula given in this answer.
    $endgroup$
    – Pierre-Yves Gaillard
    Oct 7 '11 at 15:36
















  • 1




    $begingroup$
    What did you go over in class - have you even seen these sorts of differential equations before? Were you assigned some reading material preceding the homework? Are these textbook questions that lie at the end of a chapter? It's perplexing that you'd be expected to solve an equation but not expected to learn about how to do it first...
    $endgroup$
    – anon
    Oct 7 '11 at 3:36










  • $begingroup$
    Well, in class we did this example: y''-y=delta(t-3). The homogeneous solutions were y=e^-t and y=e^t. He then deduced that the particular solution would be: [(e^t)/2]*integral from 0 to t ( (e^-s)*delta(s-3)ds ) - [e^(-t)/2]*integral from 0 to t( (e^s)*delta(s-3))ds From there, he concluded that y(t) would be e^(t-3)/2 - e^(3-t)/2 for all t>3. I guess what I'm confused on is how much of the partucluar solution came from the homogenous, and how much is just always going to be part of the particular solution
    $endgroup$
    – ben
    Oct 7 '11 at 3:53








  • 1




    $begingroup$
    And does the way he 'then deduced' the particular solution have anything to do with formulas you already have? How did he justify it to the class if not with either a derivation or a formula you're expected to know?
    $endgroup$
    – anon
    Oct 7 '11 at 4:02










  • $begingroup$
    Dear ben, you can use the formula given in this answer.
    $endgroup$
    – Pierre-Yves Gaillard
    Oct 7 '11 at 15:36










1




1




$begingroup$
What did you go over in class - have you even seen these sorts of differential equations before? Were you assigned some reading material preceding the homework? Are these textbook questions that lie at the end of a chapter? It's perplexing that you'd be expected to solve an equation but not expected to learn about how to do it first...
$endgroup$
– anon
Oct 7 '11 at 3:36




$begingroup$
What did you go over in class - have you even seen these sorts of differential equations before? Were you assigned some reading material preceding the homework? Are these textbook questions that lie at the end of a chapter? It's perplexing that you'd be expected to solve an equation but not expected to learn about how to do it first...
$endgroup$
– anon
Oct 7 '11 at 3:36












$begingroup$
Well, in class we did this example: y''-y=delta(t-3). The homogeneous solutions were y=e^-t and y=e^t. He then deduced that the particular solution would be: [(e^t)/2]*integral from 0 to t ( (e^-s)*delta(s-3)ds ) - [e^(-t)/2]*integral from 0 to t( (e^s)*delta(s-3))ds From there, he concluded that y(t) would be e^(t-3)/2 - e^(3-t)/2 for all t>3. I guess what I'm confused on is how much of the partucluar solution came from the homogenous, and how much is just always going to be part of the particular solution
$endgroup$
– ben
Oct 7 '11 at 3:53






$begingroup$
Well, in class we did this example: y''-y=delta(t-3). The homogeneous solutions were y=e^-t and y=e^t. He then deduced that the particular solution would be: [(e^t)/2]*integral from 0 to t ( (e^-s)*delta(s-3)ds ) - [e^(-t)/2]*integral from 0 to t( (e^s)*delta(s-3))ds From there, he concluded that y(t) would be e^(t-3)/2 - e^(3-t)/2 for all t>3. I guess what I'm confused on is how much of the partucluar solution came from the homogenous, and how much is just always going to be part of the particular solution
$endgroup$
– ben
Oct 7 '11 at 3:53






1




1




$begingroup$
And does the way he 'then deduced' the particular solution have anything to do with formulas you already have? How did he justify it to the class if not with either a derivation or a formula you're expected to know?
$endgroup$
– anon
Oct 7 '11 at 4:02




$begingroup$
And does the way he 'then deduced' the particular solution have anything to do with formulas you already have? How did he justify it to the class if not with either a derivation or a formula you're expected to know?
$endgroup$
– anon
Oct 7 '11 at 4:02












$begingroup$
Dear ben, you can use the formula given in this answer.
$endgroup$
– Pierre-Yves Gaillard
Oct 7 '11 at 15:36






$begingroup$
Dear ben, you can use the formula given in this answer.
$endgroup$
– Pierre-Yves Gaillard
Oct 7 '11 at 15:36












1 Answer
1






active

oldest

votes


















7












$begingroup$

The physicist's answer, not worrying about convergence, uniqueness, etc. It sounds like you have no problem away from $t=1$. Up to $1^-, y(t)=0$. Above $1$, you can solve it with $y$ being a sum of exponentials. Crossing $1$, you should integrate: $int_{1^-}^{1^+}y''+3y'=1=y'+3y|_{1^-}^{1^+}$ As $y$ can't change instantaneously, $y'$ has to go from $0$ to $1$. Then solve it starting at $t=1$ with $y=0, y'=1$






share|cite|improve this answer









$endgroup$













  • $begingroup$
    Above 1, why is this case different from below 1? Aren't both the homogeneous case and are therefore, identical?
    $endgroup$
    – Christopher Turnbull
    Nov 21 '16 at 3:52










  • $begingroup$
    @ChristopherTurnbull: Yes, away from $1$ you are back to the homogeneous case, but the initial conditions are different. We had $y'(0)=0$, but $y'(1^+)=1$. That starts a damped oscillation.
    $endgroup$
    – Ross Millikan
    Nov 21 '16 at 4:15












Your Answer





StackExchange.ifUsing("editor", function () {
return StackExchange.using("mathjaxEditing", function () {
StackExchange.MarkdownEditor.creationCallbacks.add(function (editor, postfix) {
StackExchange.mathjaxEditing.prepareWmdForMathJax(editor, postfix, [["$", "$"], ["\\(","\\)"]]);
});
});
}, "mathjax-editing");

StackExchange.ready(function() {
var channelOptions = {
tags: "".split(" "),
id: "69"
};
initTagRenderer("".split(" "), "".split(" "), channelOptions);

StackExchange.using("externalEditor", function() {
// Have to fire editor after snippets, if snippets enabled
if (StackExchange.settings.snippets.snippetsEnabled) {
StackExchange.using("snippets", function() {
createEditor();
});
}
else {
createEditor();
}
});

function createEditor() {
StackExchange.prepareEditor({
heartbeatType: 'answer',
autoActivateHeartbeat: false,
convertImagesToLinks: true,
noModals: true,
showLowRepImageUploadWarning: true,
reputationToPostImages: 10,
bindNavPrevention: true,
postfix: "",
imageUploader: {
brandingHtml: "Powered by u003ca class="icon-imgur-white" href="https://imgur.com/"u003eu003c/au003e",
contentPolicyHtml: "User contributions licensed under u003ca href="https://creativecommons.org/licenses/by-sa/3.0/"u003ecc by-sa 3.0 with attribution requiredu003c/au003e u003ca href="https://stackoverflow.com/legal/content-policy"u003e(content policy)u003c/au003e",
allowUrls: true
},
noCode: true, onDemand: true,
discardSelector: ".discard-answer"
,immediatelyShowMarkdownHelp:true
});


}
});














draft saved

draft discarded


















StackExchange.ready(
function () {
StackExchange.openid.initPostLogin('.new-post-login', 'https%3a%2f%2fmath.stackexchange.com%2fquestions%2f70512%2fsolving-a-differential-equation-with-the-dirac-delta-function-without-laplace-tr%23new-answer', 'question_page');
}
);

Post as a guest















Required, but never shown

























1 Answer
1






active

oldest

votes








1 Answer
1






active

oldest

votes









active

oldest

votes






active

oldest

votes









7












$begingroup$

The physicist's answer, not worrying about convergence, uniqueness, etc. It sounds like you have no problem away from $t=1$. Up to $1^-, y(t)=0$. Above $1$, you can solve it with $y$ being a sum of exponentials. Crossing $1$, you should integrate: $int_{1^-}^{1^+}y''+3y'=1=y'+3y|_{1^-}^{1^+}$ As $y$ can't change instantaneously, $y'$ has to go from $0$ to $1$. Then solve it starting at $t=1$ with $y=0, y'=1$






share|cite|improve this answer









$endgroup$













  • $begingroup$
    Above 1, why is this case different from below 1? Aren't both the homogeneous case and are therefore, identical?
    $endgroup$
    – Christopher Turnbull
    Nov 21 '16 at 3:52










  • $begingroup$
    @ChristopherTurnbull: Yes, away from $1$ you are back to the homogeneous case, but the initial conditions are different. We had $y'(0)=0$, but $y'(1^+)=1$. That starts a damped oscillation.
    $endgroup$
    – Ross Millikan
    Nov 21 '16 at 4:15
















7












$begingroup$

The physicist's answer, not worrying about convergence, uniqueness, etc. It sounds like you have no problem away from $t=1$. Up to $1^-, y(t)=0$. Above $1$, you can solve it with $y$ being a sum of exponentials. Crossing $1$, you should integrate: $int_{1^-}^{1^+}y''+3y'=1=y'+3y|_{1^-}^{1^+}$ As $y$ can't change instantaneously, $y'$ has to go from $0$ to $1$. Then solve it starting at $t=1$ with $y=0, y'=1$






share|cite|improve this answer









$endgroup$













  • $begingroup$
    Above 1, why is this case different from below 1? Aren't both the homogeneous case and are therefore, identical?
    $endgroup$
    – Christopher Turnbull
    Nov 21 '16 at 3:52










  • $begingroup$
    @ChristopherTurnbull: Yes, away from $1$ you are back to the homogeneous case, but the initial conditions are different. We had $y'(0)=0$, but $y'(1^+)=1$. That starts a damped oscillation.
    $endgroup$
    – Ross Millikan
    Nov 21 '16 at 4:15














7












7








7





$begingroup$

The physicist's answer, not worrying about convergence, uniqueness, etc. It sounds like you have no problem away from $t=1$. Up to $1^-, y(t)=0$. Above $1$, you can solve it with $y$ being a sum of exponentials. Crossing $1$, you should integrate: $int_{1^-}^{1^+}y''+3y'=1=y'+3y|_{1^-}^{1^+}$ As $y$ can't change instantaneously, $y'$ has to go from $0$ to $1$. Then solve it starting at $t=1$ with $y=0, y'=1$






share|cite|improve this answer









$endgroup$



The physicist's answer, not worrying about convergence, uniqueness, etc. It sounds like you have no problem away from $t=1$. Up to $1^-, y(t)=0$. Above $1$, you can solve it with $y$ being a sum of exponentials. Crossing $1$, you should integrate: $int_{1^-}^{1^+}y''+3y'=1=y'+3y|_{1^-}^{1^+}$ As $y$ can't change instantaneously, $y'$ has to go from $0$ to $1$. Then solve it starting at $t=1$ with $y=0, y'=1$







share|cite|improve this answer












share|cite|improve this answer



share|cite|improve this answer










answered Oct 9 '11 at 3:28









Ross MillikanRoss Millikan

300k24200375




300k24200375












  • $begingroup$
    Above 1, why is this case different from below 1? Aren't both the homogeneous case and are therefore, identical?
    $endgroup$
    – Christopher Turnbull
    Nov 21 '16 at 3:52










  • $begingroup$
    @ChristopherTurnbull: Yes, away from $1$ you are back to the homogeneous case, but the initial conditions are different. We had $y'(0)=0$, but $y'(1^+)=1$. That starts a damped oscillation.
    $endgroup$
    – Ross Millikan
    Nov 21 '16 at 4:15


















  • $begingroup$
    Above 1, why is this case different from below 1? Aren't both the homogeneous case and are therefore, identical?
    $endgroup$
    – Christopher Turnbull
    Nov 21 '16 at 3:52










  • $begingroup$
    @ChristopherTurnbull: Yes, away from $1$ you are back to the homogeneous case, but the initial conditions are different. We had $y'(0)=0$, but $y'(1^+)=1$. That starts a damped oscillation.
    $endgroup$
    – Ross Millikan
    Nov 21 '16 at 4:15
















$begingroup$
Above 1, why is this case different from below 1? Aren't both the homogeneous case and are therefore, identical?
$endgroup$
– Christopher Turnbull
Nov 21 '16 at 3:52




$begingroup$
Above 1, why is this case different from below 1? Aren't both the homogeneous case and are therefore, identical?
$endgroup$
– Christopher Turnbull
Nov 21 '16 at 3:52












$begingroup$
@ChristopherTurnbull: Yes, away from $1$ you are back to the homogeneous case, but the initial conditions are different. We had $y'(0)=0$, but $y'(1^+)=1$. That starts a damped oscillation.
$endgroup$
– Ross Millikan
Nov 21 '16 at 4:15




$begingroup$
@ChristopherTurnbull: Yes, away from $1$ you are back to the homogeneous case, but the initial conditions are different. We had $y'(0)=0$, but $y'(1^+)=1$. That starts a damped oscillation.
$endgroup$
– Ross Millikan
Nov 21 '16 at 4:15


















draft saved

draft discarded




















































Thanks for contributing an answer to Mathematics Stack Exchange!


  • Please be sure to answer the question. Provide details and share your research!

But avoid



  • Asking for help, clarification, or responding to other answers.

  • Making statements based on opinion; back them up with references or personal experience.


Use MathJax to format equations. MathJax reference.


To learn more, see our tips on writing great answers.




draft saved


draft discarded














StackExchange.ready(
function () {
StackExchange.openid.initPostLogin('.new-post-login', 'https%3a%2f%2fmath.stackexchange.com%2fquestions%2f70512%2fsolving-a-differential-equation-with-the-dirac-delta-function-without-laplace-tr%23new-answer', 'question_page');
}
);

Post as a guest















Required, but never shown





















































Required, but never shown














Required, but never shown












Required, but never shown







Required, but never shown

































Required, but never shown














Required, but never shown












Required, but never shown







Required, but never shown







Popular posts from this blog

Can a sorcerer learn a 5th-level spell early by creating spell slots using the Font of Magic feature?

Does disintegrating a polymorphed enemy still kill it after the 2018 errata?

A Topological Invariant for $pi_3(U(n))$